diff --git a/Exam/M61B_2023-24_DS2-solutions.pdf b/Exam/M61B_2023-24_DS2-solutions.pdf new file mode 100644 index 0000000000000000000000000000000000000000..f1aecb431d7dac87c314e40c539ddfbcaea2e4d3 Binary files /dev/null and b/Exam/M61B_2023-24_DS2-solutions.pdf differ diff --git a/Exam/M61B_2023-24_DS2-sujet.pdf b/Exam/M61B_2023-24_DS2-sujet.pdf new file mode 100644 index 0000000000000000000000000000000000000000..a663bea855f71e9a463c57d490359b95bb4135a8 Binary files /dev/null and b/Exam/M61B_2023-24_DS2-sujet.pdf differ diff --git a/Exam/M61B_2023-24_DS2.tex b/Exam/M61B_2023-24_DS2.tex new file mode 100644 index 0000000000000000000000000000000000000000..9abb42650b66aa095ea2a87dab6e28d79d0f62cb --- /dev/null +++ b/Exam/M61B_2023-24_DS2.tex @@ -0,0 +1,388 @@ +\documentclass[a4paper,12pt,reqno]{amsart} +\usepackage{lille} +\lilleset{ + % solutions, + % sans enonces, + titre=\sisujet{Examen final}\sisolutions{Solutions de l'examen}, + date=18 avril 2024, + duree=3 heures, +} +\input{m61proba} + +\begin{document} +\sisujet{ + \tsvp + \vspace{7mm} + \attention~ + \emph{Les documents et les objets électroniques sont interdits. Les exercices sont indépendants. Toutes les réponses doivent être justifiées.} + \vspace{7mm} +} + +% ----------------------------------------------- 1 +\begin{exo} + + Soient $X$ et $Y$ deux variables aléatoires indépendantes. On suppose que $X$ suit la loi géométrique de paramètre $\alpha$ et que $Y$ suit la loi géométrique de paramètre $\beta$, avec $\alpha,\beta\in ]0, 1[$. On note + \[ + Z \coloneqq \min(X, Y). + \] + \begin{enumerate} + \item Pour tout $k\in\N$, calculer $\PP{X > k}$ et $\PP{Y > k}$. + \item Calculer $\PP{Z > k}$, puis en déduire la loi de $Z$. Identifier cette loi. + \end{enumerate} +\end{exo} + +\begin{solution} + + \begin{enumerate} + \item Pour tout $k\in\N$, on a + \[ + \PP{X > k} + = \sum_{j > k} \PP{X = j} + = \sum_{j \geq k+1} \alpha(1-\alpha)^{j-1} + = \alpha(1-\alpha)^k\frac{1}{1-(1-\alpha)} + = (1-\alpha)^k, + \] + en utilisant la somme de la série géométrique de premier terme $\alpha(1-\alpha)^k$ et de raison $(1-\alpha)\in]0,1[$. De même, on trouve + \[ + \PP{Y > k} = (1-\beta)^k. + \] + + \item Pour tout $k\in\N$, on a + \[ + \PP{Z > k} + = \PP{X > k,\, Y > k} + = \PP{X>k}\times\PP{Y > k} + = ((1-\alpha)(1-\beta))^k + \] + par indépendance de $X$ et $Y$. En posant $\gamma \coloneqq \alpha + \beta - \alpha\beta$ on trouve $\PP{Z > k} = (1-\gamma)^k$. Ainsi, pour tout $k\geq 1$, + \[ + \PP{Z = k} + = \PP{Z > k-1} - \PP{Z > k} + = \gamma(1-\gamma)^{k-1}. + \] + Comme $Z = \min(X,Y)\geq 1$, vu que $X\geq 1$ et $Y\geq 1$, ceci signifie que $Z$ suit une loi géométrique de paramètre $\gamma$. On vérifie aussi que $\gamma\in ]0,1[$, puisque $\alpha,\beta\in ]0,1[$. + \end{enumerate} + +\end{solution} + + +% ----------------------------------------------- 2 +\begin{exo} + + Soient $\mu \in \R$ et $\sigma \in \R^{*}$. Une variable aléatoire $X$ à valeurs dans $]0,+\infty[$ est dite suivre une loi \emph{log-normale} de paramètres $(\mu, \sigma^2)$ si $Y = \log X$ suit la loi gaussienne $\Nor{\mu,\sigma^2}$. On note $\LogNor{\mu,\sigma^2}$ la loi log-normale de paramètres $(\mu, \sigma^2)$. + \begin{enumerate} + \item Exprimer $X$ à l'aide d'une variable $Z$ (à préciser) qui suit une loi normale centrée réduite. + En déduire la fonction de répartition de $X$ à l'aide de la fonction $\Phi$ de répartition de $Z$. + Calculer ensuite explicitement la densité de $X$. + \item Calculer l'espérance de $X$. + \item Montrer que $X^r\sim\LogNor{r\mu,r^2\sigma^2}$ pour tout $r \neq 0$. En déduire la valeur de $\EE{X^r}$ pour tout $r\in\R$. Calculer la variance de $X$. + \item Calculer $\EE{e^{uX}}$ pour tout $u >0$. + \end{enumerate} +\end{exo} + +\begin{solution} + \begin{enumerate} + \item\label{s2a} On a $X = e^Y$ avec $Y\sim\Nor{\mu, \sigma^2}$. En posant $Z\coloneqq \frac{Y-\mu}{\sigma}$ on trouve $X = e^{\mu +\sigma Z}$ avec $Z\sim\Nor{0, 1}$. Comme $X\geq0$, on en déduit $\PP{X\leq x} = 0$ pour tout $x\leq 0$, et pour tout $x > 0$ on a + \[ + \PP{X \leq x} + = \PP{Z \leq \frac{\log x - \mu}{\sigma}} + = \Phi\pa{\frac{\log x - \mu}{\sigma}}. + \] + En dérivant on trouve que la densité de $X$ vaut + \[ + \rho_X(x) + = \rho_Z\pa{\frac{\log x - \mu}{\sigma}}\frac{1}{\sigma x} + = \frac{1}{\sigma x \sqrt{2\pi}} \exp\left[ -\frac{(\log x -\mu)^2}{2\sigma^2}\right] + \] + sur $]0,\infty[$ et zéro ailleurs. + + \item\label{s2b} En utilisant le théorème de transfert, on trouve + \[ + \EE{X} + = \EE{e^{\mu +\sigma Z}} + = e^{\mu}\tfrac{1}{\sqrt{2\pi}}\int_{-\infty}^\infty e^{\sigma t} e^{-\frac{t^2}{2}} \dd t + = e^{\mu + \frac{\sigma^2}{2}}\tfrac{1}{\sqrt{2\pi}}\int_{-\infty}^\infty e^{-\frac{(t-\sigma)^2}{2}} \dd t + = e^{\mu + \frac{\sigma^2}{2}}. + \] + + \item On a + \[ + X^r = e^{r\mu + (r\sigma) Z} + \] + et ceci entraîne par \eqref{s2a} que $X^r\sim\LogNor{r\mu,r^2\sigma^2}$ pour tout $r \neq 0$, et donc par \eqref{s2b} que + \[ + \EE{X^r} = e^{r\mu + r^2\sigma^2/2} + \] + pour tout $r\in\R$. On en déduit la variance de $X$ + \[ + \VV{X} + = \EE{X^2} - \bpa{\EE{X}}^2 + = e^{2\mu + 2\sigma^2} - e^{2\mu + \sigma^2} + = e^{2\mu + \sigma^2} (e^{\sigma^2} -1). + \] + + \item On a par le \eqref{s2a}, et en appliquant le théorème de transfert, + \[ + \EE{e^{uX}} + = \EE{e^{ue^{\mu + \sigma Z}}} + = \frac{1}{\sqrt{2\pi}}\int_\R e^{ue^{\mu + \sigma x} - x^2/2} \dd x + = \infty + \] + puisque $u, \sigma > 0$ et donc $e^{ue^{\mu + \sigma x} - x^2/2}\to\infty$ quand $x\to\infty$. + + \end{enumerate} +\end{solution} + + + +% ----------------------------------------------- 3 +\begin{exo} + + \emph{Les trois questions dans cet exercice sont indépendantes les unes des autres.} + \begin{enumerate} + \item Montrer que + \[ + \sum_{n\geq 0} e^{-nx} = \frac{1}{1-e^{-x}} + \] + pour tout $x > 0$. En appliquant le théorème de convergence monotone, en déduire que + \[ + \int_0^\infty \frac{x e^{-x}}{1-e^{-x}} \dd x = \sum_{n\geq 1} \frac{1}{n^2}\cdot + \] + Connaissez-vous la valeur de cette série ? + + \item En utilisant le théorème de convergence dominée, calculer explicitement + \[ + \lim_{n\to\infty} \pa{\int_0^\infty \frac{n^2 \sin(x/n)}{(1+nx)(1+x^2)} \dd x}. + \] + + \medskip + + \item En utilisant le lemme de Fatou, montrer que + \[ + \lim_{n\to\infty} \pa{ \int_0^\infty \frac{n}{\sin^2 n + nx^2} \dd x} = \infty. + \] + \end{enumerate} + +\end{exo} + +\begin{solution} + \begin{enumerate} + \item Comme $x > 0$ on a $e^{-x} \in ]0,1[$ et donc + \[ + \sum_{n\geq 0} e^{-nx} = \frac{1}{1-e^{-x}} + \] + par la somme de la série géométrique. On en déduit + \[ + \int_0^\infty \frac{x e^{-x}}{1-e^{-x}} \dd x + = \int_0^\infty \sum_{n\geq 0} x e^{-x}e^{-nx} \dd x + = \int_0^\infty \lim_{n\to\infty}\sum_{k= 0}^n x e^{-x}e^{-kx} \dd x + \] + Comme la série est à termes positifs, la suite des sommes partielles est positive est croissante, donc on peut appliquer le théorème de convergence monotone qui entraîne + \[ + \int_0^\infty \frac{x e^{-x}}{1-e^{-x}} \dd x + = \lim_{n\to\infty}\int_0^\infty \sum_{k= 0}^n x e^{-(k+1)x} \dd x + = \lim_{n\to\infty} \sum_{k= 0}^n \pa{ \int_0^\infty x e^{-(k+1)x} \dd x}. + \] + Comme + \[ + \int_0^\infty x e^{-(k+1)x} \dd x + = \frac{1}{(k+1)^2}\int_0^\infty y e^{-y} dy + = \frac{1}{(k+1)^2} + \] + pour tout $k\geq 0$ en ayant fait le changement de variable $y = (k+1) x$, on trouve finalement + \[ + \int_0^\infty \frac{x e^{-x}}{1-e^{-x}} \dd x + = \lim_{n\to\infty} \sum_{k= 0}^n \frac{1}{(k+1)^2} + = \sum_{n\geq 1} \frac{1}{n^2}\cdot + \] + Et oui, nous connaissons bien la valeur de cette série qui est $\pi^2/6$ (problème de Bâle). + + \item On a + \[ + \frac{n^2 \sin(x/n)}{(1+nx)(1+x^2)} + = \frac{nx}{1+nx}\times\frac{\sin(x/n)}{(x/n)}\times\frac{1}{1+x^2}\to \frac{1}{1+x^2} + \] + quand $n\to\infty$ pour tout $x > 0$ puisque $(\sin z)/z\to 1$ quand $z\to 0$. D'autre part on a $\vert (\sin (x/n))/(x/n)\vert < 1$ et $nx/(1+nx) < 1$ pour tout $x > 0$ et $n\geq 0$, d'où + \[ + \abs{\frac{n^2 \sin(x/n)}{(1+nx)(1+x^2)}} < \frac{1}{1+x^2} + \] + pour tout $x > 0$ et $n\geq 0$, et la fonction de droite est intégrable. Par le théorème de convergence dominée, on obtient + \[ + \lim_{n\to\infty} \pa{\int_0^\infty \frac{n^2 \sin(x/n)}{(1+nx)(1+x^2)} \dd x} + = \int_0^\infty \frac{1}{1+x^2} \dd x + = \frac{\pi}{2}\cdot + \] + + \item On a + \[ + \frac{n}{\sin^2 n + nx^2} = \frac{1}{x^2 + (\sin^2 n/n)} \to \frac{1}{x^2} + \] + quand $n\to\infty$ et donc + \[ + \liminf_{n\to\infty}\pa{ \frac{n}{\sin^2 n + nx^2}} = \frac{1}{x^2} + \] + pour tout $x > 0$. + Par le lemme de Fatou, on a + \[ + \liminf_{n\to\infty} \pa{ \int_0^\infty \frac{n}{\sin^2 n + nx^2} \dd x} + \geq \int_0^\infty \liminf_{n\to\infty} \pa{\frac{n}{\sin^2 n + nx^2}} \dd x + = \int_0^\infty \frac{dx}{x^2} + = \infty + \] + et donc + \[ + \lim_{n\to\infty} \pa{ \int_0^\infty \frac{n}{\sin^2 n + nx^2} \dd x} + = \infty. + \] + \end{enumerate} +\end{solution} + + +% ----------------------------------------------- 4 +\begin{exo} + + Soit $\suite[n\geq 1]{X_n}$ une suite de variables aléatoires suivant une loi exponentielle de paramètre 1, dont on rappelle que la densité est donnée par $e^{-x}\un{x >0}$ sur $\R$. On suppose que les $X_n$ sont mutuellement indépendantes. On note $M_0 = 0$ et + \[ + M_n \coloneqq \max (X_1, \ldots, X_n), \qquad n\geq 1. + \] + \begin{enumerate} + \item Calculer la fonction de répartition $F_n (x) = \PP{M_n \leq x}$ pour tout $n\geq 1$ et $x\in\R$. En déduire $G_n (x) \coloneqq \PP{M_n - \log n \leq x}$ pour tout $n\geq 1$ et $x\in\R$. + + \item\label{4b} Calculer la fonction de répartition de $Z_1 = -\log X_1$. En déduire à l'aide de la question précédente et d'un équivalent approprié que + \[ + M_n - \log n \darrow -\log X_1 + \] + quand $n\to \infty$. + + \item En utilisant les fonctions $F_n$ et $F_{n-1}$, montrer que + \[ + \EE{M_n} = \EE{M_{n-1}} + \frac{1}{n} + \] + pour tout $n\geq 1$. En déduire que + \[ + \EE{M_n} = 1 + \frac{1}{2} + \cdots + \frac{1}{n}\cdot + \] + \indication{On rappelle la formule $\EE{X} = \int_{0}^{\infty}\pa{1-F_X(t)} \dd t - \int_{-\infty}^{0}F_X(t) \dd t$, où $F_{X}$ est la fonction de répartition de $X$.} + + \item En utilisant le théorème de convergence dominée et la question \eqref{4b}, montrer que + \[ + \EE{M_n} - \log n \to \EE{-\log X_1} + \] + quand $n\to\infty$.\\ + \indication{On rappelle que $\log(1-z)\leq -z$ pour tout $z < 1$ et on pourra vérifier que $\log(1-z)\geq -2z$ pour tout $z\in [0,1/2]$.} + + \item En utilisant tout ce qui précède, montrer que la suite + \[ + u_n = 1 + \frac{1}{2} + \cdots + \frac{1}{n} - \log n + \] + converge quand $n\to\infty$ vers une constante finie, que l'on exprimera sous forme intégrale. Connaissez-vous le nom de cette constante ? Est-elle un nombre rationnel ? + \end{enumerate} +\end{exo} + +\begin{solution} + \begin{enumerate} + \item Rappelons que la fonction de répartition de la loi exponentielle est $F(x) = \pa{1 - e^{-x}}_+$, avec la notation $z_+ \coloneqq \max(0,z)$ pour tout $z\in\R$. + Pour tout $n\geq 1$, on trouve + \[ + \PP{M_n \leq x} + = \PP{X_1\leq x, \ldots, X_n\leq x} + = \PP{X_1 \leq x} \times\cdots\times\PP{X_n \leq x} + = (1-e^{-x})_+^n. + \] + On en déduit, pour tout $n\geq 1$ et $x\in\R$, + \[ + G_n (x) + = \PP{M_n \leq x + \log n} + = \pa{1 - \frac{e^{-x}}{n}}_+^n. + \] + \item\label{s4b} Pour tout $x\in\R$, on calcule + \[ + \PP{Z_1 \leq x} + = \PP{\log X_1 \geq -x} + = \PP{X_1 \geq e^{-x}} = e^{-e^{-x}}, + \] + car $\PP{X_1 \geq \alpha} = \PP{X_1 > \alpha} = e^{-\alpha}$ pour $\alpha \geq 0$. On en déduit que + \[ + G_n (x) + = \pa{1 - \frac{e^{-x}}{n}}_+^n + = \exp\left[n \log\pa{1 - \frac{e^{-x}}{n}}\right] + \] + pour tout $x > -\log n$ et donc, à l'aide de l'équivalent $\log(1-z)\sim z$ quand $z\to 0$ que + \[ + \PP{M_n - \log n \leq x} \to e^{-e^{-x}} + = \PP{Z_1 \leq x} + \] + quand $n\to \infty$ pour tout $x\in\R$ puisqu'à partir d'un certain rang on aura $x \geq -\log n\to -\infty$ quand $n\to \infty$. Ceci entraîne par définition que + \[ + M_n - \log n \darrow -\log X_1 + \] + quand $n\to \infty$. + + \item Comme $M_n$ est une variable aléatoire positive, on peut utiliser la formule + \[ + \EE{M_n} = \int_0^\infty \PP{M_n > x} dx + = \int_0^\infty (1- F_n(x)) dx + = \int_0^\infty \pa{1 - (1- e^{-x})^n} dx + \] + pour tout $n\geq 1$. Comme + \[ + 1 - (1- e^{-x})^n + = 1 - (1- e^{-x})^{n-1} + e^{-x} (1- e^{-x})^{n-1} + \] + on obtient + \[ + \EE{M_n} + = \EE{M_{n-1}} + \int_0^\infty e^{-x} (1- e^{-x})^{n-1} dx + = \EE{M_{n-1}} + \int_0^1 (1- z)^{n-1} dz + \] + en faisant le changement de variable $z = e^{-x}$, ce qui donne + \[ + \EE{M_n} + = \EE{M_{n-1}} + \frac{1}{n} + \] + et donc + \[ + \EE{M_n} \; + = 1 + \frac{1}{2} + \cdots + \frac{1}{n} + \] + par itération, puisque $\EE{M_1} =\EE{X_1} = 1$. + + \item On a + \[ + \EE{M_n} - \log n + = \EE{M_n -\log n} + = \int_0^\infty (1-G_n(x)) dx - \int_{-\infty}^0 G_n(x) dx + \] + par la formule du cours sur l'espérance d'une variable aléatoire réelle. + Par \eqref{s4b} on a $G_n(x)\to G(x) = \PP{-\log X_1 \leq x}$ pour tout $x\in\R$ et sous les hypothèses du théorème de convergence dominée on aura donc + \[ + \EE{M_n} - \log n \to \int_0^\infty (1-G(x)) dx - \int_{-\infty}^0 G(x) dx + = \EE{-\log X_1} + \] + quand $n\to\infty$ comme requis. Reste à vérifier les hypothèses du théorème de convergence dominée. On sait que $\log(1-z)\leq -z$ pour tout $z < 1$ et donc + \[ + G_n(x) + = \exp\left[n \log\pa{1 - \frac{e^{-x}}{n}}_+\right] \leq e^{-e^{-x}} + \] + pour tout $x\geq 0$, et le terme de droite est une fonction intégrable sur $(-\infty,0)$. On vérifie aussi que $1- z \geq e^{-2z}$ pour tout $z\in [0,1/2]$ puisque la fonction $z\mapsto 1 -z -e^{-2z}$ est nulle en $z=0$, strictement positive en $z=1/2$ et de dérivée $2e^{-2z} -1$ qui est positive puis négative. Ceci entraîne par croissance du logarithme + \[ + 1- G_n(x) + = 1 - \exp\left[n \log\pa{1 - \frac{e^{-x}}{n}}\right] \leq 1- e^{-2e^{-x}} + \] + pour tout $x\geq 0$ et $n\geq 2$, avec $1- e^{-2e^{-x}} \leq 2e^{-x}$ qui est bien une fonction intégrable sur $]0,\infty[$. + + \item En utilisant tout ce qui précède, on voit que + \[ + u_n = 1 + \frac{1}{2} + \cdots + \frac{1}{n} - \log n \to \;\EE{-\log X_1} + \] + quand $n\to\infty$. Et par la formule de transfert on a + \[ + \EE{-\log X_1} + = \int_0^\infty (-\log x) e^{-x} dx + \] + qui est bien une intégrale convergente en zéro et en l'infini. Il s'agit de la constante $gamma$ d'Euler, dont on ne sait pas si elle est rationnelle ou non (mais on pense que non). + \end{enumerate} +\end{solution} + +\end{document} + diff --git a/Exam/m61proba.tex b/Exam/m61proba.tex index 7c0b55ec0654adf805adf296ea8894c67c31476f..2a4d8365b3cf5538a2ebd2948ef40535c46998be 100644 --- a/Exam/m61proba.tex +++ b/Exam/m61proba.tex @@ -25,6 +25,11 @@ \newcommand{\1}{\mathds{1}} \newcommand{\un}[1]{\mathbf{1}_{\{#1\}}} +% Macros Thomas +\newcommand*{\deq}{\stackrel{d}{=}} +\newcommand*{\darrow}{\xrightarrow{\kern3pt d\kern4pt}} +\newcommand*{\parrow}{\stackrel{\kern3pt (\P)\kern4pt}{\rightarrow}} + % Macros Kroum \newcommand*{\suite}[2][n]{\left( #2 \right)_{#1}}% exemple \suite{u_n}, \suite[k>0]{u_k} \newcommand*{\suiteN}[1]{\suite[n\in\N]{#1}}% exemple \suiteN{u_n} @@ -53,6 +58,8 @@ \newcommand{\Bin}[1]{\operatorname{Bin}(#1)}% loi binomiale \newcommand{\Exp}[1]{\mathcal{E}(#1)}% loi exponentielle \newcommand{\Uni}[1]{\mathcal{U}(#1)}% loi uniforme +\newcommand{\Nor}[1]{\mathcal{N}(#1)}% loi normale +\newcommand{\LogNor}[1]{\mathcal{L}(#1)}% loi log-normale % indications \newcommand{\indication}[1]{\emph{Indication : #1}} diff --git a/README.md b/README.md index 8a346602c3fbcb8c3a6312ef7e2cdd9f7bc8203b..98ed53466e674a41678a0ef1db5af92cfb22e8a8 100644 --- a/README.md +++ b/README.md @@ -19,6 +19,7 @@ Dans [ce dépôt](https://gitlab.univ-lille.fr/tzanev/l3m61proba/) vous pouvez t ### Les sujets d'examens et leurs solutions - DS1 **[[sujet](Exam/M61B_2023-24_DS1-sujet.pdf)]** **[[solutions](Exam/M61B_2023-24_DS1-solutions.pdf)]** [[tex](Exam/M61B_2023-24_DS1.tex)] +- DS2 **[[sujet](Exam/M61B_2023-24_DS2-sujet.pdf)]** **[[solutions](Exam/M61B_2023-24_DS2-solutions.pdf)]** [[tex](Exam/M61B_2023-24_DS2.tex)] ### Source complémentaires diff --git a/TDs/m61proba.tex b/TDs/m61proba.tex index 37319f9c013b756b99c8d472dbf0b5422b6076a8..2a4d8365b3cf5538a2ebd2948ef40535c46998be 100644 --- a/TDs/m61proba.tex +++ b/TDs/m61proba.tex @@ -25,12 +25,18 @@ \newcommand{\1}{\mathds{1}} \newcommand{\un}[1]{\mathbf{1}_{\{#1\}}} +% Macros Thomas +\newcommand*{\deq}{\stackrel{d}{=}} +\newcommand*{\darrow}{\xrightarrow{\kern3pt d\kern4pt}} +\newcommand*{\parrow}{\stackrel{\kern3pt (\P)\kern4pt}{\rightarrow}} + % Macros Kroum \newcommand*{\suite}[2][n]{\left( #2 \right)_{#1}}% exemple \suite{u_n}, \suite[k>0]{u_k} \newcommand*{\suiteN}[1]{\suite[n\in\N]{#1}}% exemple \suiteN{u_n} \newcommand*{\abs}[1]{\left\lvert{\ifx\hfuzz#1\hfuzz \,\cdot\,\else#1\fi}\right\rvert}% |.| \newcommand*{\norm}[1]{\left\lVert{\ifx\hfuzz#1\hfuzz \,\cdot\,\else#1\fi}\right\rVert}% norme \newcommand*{\scalprod}[3][]{#1\langle{#2}\kern1pt #1|{#3}#1\rangle}% exemple \scalprod[\big]{A}{B} +\newcommand*{\ens}[2][]{#1\{ #2 #1\}}% exemple \ens[\big]{A,B,C} \newcommand*{\ensemble}[3][]{#1\{ #2 \;#1|\; #3 #1\}}% exemple \ensemble[\big]{x^2}{x \in \R} \newcommand{\iintv}[1]{\ldbrack #1\rdbrack}% intervalle d'entiers \newcommand{\dd}{\,\mathrm{d}}% le «d» du dx, dt, ... @@ -52,6 +58,8 @@ \newcommand{\Bin}[1]{\operatorname{Bin}(#1)}% loi binomiale \newcommand{\Exp}[1]{\mathcal{E}(#1)}% loi exponentielle \newcommand{\Uni}[1]{\mathcal{U}(#1)}% loi uniforme +\newcommand{\Nor}[1]{\mathcal{N}(#1)}% loi normale +\newcommand{\LogNor}[1]{\mathcal{L}(#1)}% loi log-normale % indications \newcommand{\indication}[1]{\emph{Indication : #1}}